Вы находитесь на странице: 1из 15

Logical Reasoning - Puzzles

In the recent past exams, CAT verbal ability section consisted of about 10 logical
reasoning puzzle questions. This type of questions typically has a set of facts and
some rules which apply on those facts. The question offers different choices, one
of which follows from these facts and rules. Sometimes the question also specifies
some facts and rules and one has to consider them also in arriving at the
solution..

A methodical approach to a reasoning question makes life a lot easier for the
students. We have outlined below a general approach to solve these logical
reasoning puzzles and have also illustrated them with some actual questions that
appeared on the CAT.

Steps for solving logical reasoning puzzles

Step 1: Write down the constraints and other facts given in the problem
succinctly. See if there are additional facts that can be derived based on the
above.

Step 2: Use the information in the question which would be specific to that
question and add it to the general constraints.

Step 3: Take each choice, assume it to be correct and see whether it either
violates a constraint or satisfies all the constraints noted down in step 1 and step
2. Sometimes the rules have to be applied till a constraint is violated or all the
constraints are satisfied.

If even after application of the rules, there are many possibilities for arriving at a
solution for a choice, take each of such possibility and see if either each of them
violates the constraint or satisfies the constraints. Then either accept or eliminate
the choice .

Depending on what is asked in the question, the choice is not correct if either any
of the constraints are violated or all the constraints are satisfied. If the choice is
not correct, go on to the next choice and repeat this step.

Stop at the right choice.


Note: Use info from previously solved questions in a puzzle to answer the current question, provided
the same conditions hold for the current question .

Illustrations from CAT 2012 Exam


Questions 48-51

Two teams of five each must be selected from a group of ten persons a through J
of which A,E,G are doctors; D,H,J are lawyers; B and I are engineers; C and F are
managers. It is known that,

(i) every team must contain persons from each of the four professions
(ii) C and H cannot be selected together
(iii) I cannot be selected into a team with 2 lawyers

(iv) J cannot be in a team with 2 doctors

(iv) A and D cannot be selected together

Solution:

Step 1: Write down the facts and rules

A,E, G – Doctors

D, H, J – Lawyers

B, I – Engineers
C, F – Managers

A and D different groups

C and H different groups

Each group should contain a Doc, Law, Eng and Man

I not in the same group as 2 Law

J not in the same group as 2 Doc

Derived Facts :

B and I not in the same group

C and F not in the same group

48. If C and G are in different teams, who are the other team members of A?

a. C,D,E , I
b. B,F, I, J
c. B,C,H,J
d. F,H,I,G

Step 2: Use the info in the question

Step 3: In addition, use the constraints given in the problem

G, F, H

Step 4: Add to the above the info given in each choice 1 by 1


Choice a:

C,A, D, E,I – Not correct because it violates A and D should be in different groups

Choice b:

C,A,B,F,I,J – Violates the constraint that there can be only 5 members in a team.

Choice c:

C,A,B,H,J- Violates the constraint that c and H should be in different groups.

The remaining choice d has to be correct. Let us test it.

A,F,H,I,G

C, D,E,B,J

Correct choice because it doesn’t violate any constraint.

Hence the correct answer is Choice d.

49. Who among the following cannot be in the same team as I?

a. H

b. J

c. C

d. F

We can use the info from question 48

We see that we can rule out choices a and d.

Step 2: use info in the question:

Step 3: Use info in the problem.


I

Step 4: Use info in choices b and c.

Choice b:

I, J,

Use the possibilities to proceed towards the solution:

They are: I,J, (A/D),(C/HF), (G/E)

Expand them and see if you can arrive at the solution

The possibilities are:

1. IJAC(G/E)
2. IJDC(G,E)
3. IJAHF
4. IJDHF

Violation occurs in all of the above. So J cannot be in the same team as I.

Hence the correct answer is choice b.

50. Who among the following must always be in the same team as A?

a. D

b. B

c. H

d. J

We can see from answer of Q.48 that none of the choices given in Q.50 except H
is with A. So the correct answer is Choice c.
51.If F and G are in the same team , which amnong the following statements is
true?

a. B and H will be in the other team.

b. E and I must be in the same team.

c. H must be in the same team but B must be in the other team.

d. c must be in the other team but D must be in the same team.

Again referring to the answer of Q.48, we can see that only choice c of Q.51 is
correct.

Note: If one cannot use information from the previous questions, one has to
solve it in the normal way

Questions 52-55

Two families are planning to go on a canoe trip together. The families consists of
the following people: Robert and Mary Henderson and their three sons Tommy,
Don and William. Jerome and Ellen Penick and their two daughters Susan and
Kate.

There will be three canoes with three people in each canoe. At least one of the
four parents must be in each canoe. At least one person from each family must be
in each canoe.

Solution:

Step 1:Write down the facts and the rules in the problem

Family 1- R,M,T,D,W

Family 2- J,E,K,S

One of R,M,J,E should be in each group


One of Family 1 and Family 2 should be in each group

52. If the two mothers ride together in the same canoe and the three brothers
each ride in a different canoe, which of the following must be true?

a. Each canoe has both males and females in it

b. One of the canoes has only females in it

c. One of the canoes has only males in it.

d. The sisters ride in the same canoe.

Step 2: Use info in the question

E,M,T

Step 3: Use info in the problem

E,M, T/D/W

R/J, K/S, T/D/W

R/J, K/S , T/D/W

Step 4: Use info in choices 1 by 1. Not really required because step 3 itself gives
adequate info.

Choice a: Correct

Other choices are not correct.

53. If Ellen and Susan are together in one of the canoes, which of the following
could be a list of people together in another canoe?

a. Dan, Jerome, Kate

b.Dan, Jerome, William


c. Dan, Kate, Tommy

d. Jerome, Kate, Mary

Step 2:Use info in the question.

E,S

Step 3: Use info in the problem

E,S

There is nothing we can definitively add

Step 4: Use info in choices 1 by 1

Choice a:

E,S, T/W

D,J,K

R/M, T/W, T/W

We see that the third group will contain only Family 1 and is a violation

Choice b:

E,S, T/K

D,J,W

R/M, T/K, T/K

No violation. Hence is the correct answer.

54. If Jerome and Mary are together in one of the canoes each of the following
could be a list of people in another canoe except,

a. Dan, Ellen, Susan

b. Ellen, Robery, Tommy


c. Ellen,Susan, William

d. Ellen, Tommy, William

Step 2: Use info in the question

J,M

Step 3: Nothing definitive can be used from the problem.

Step 4: Use info from the choices 1 by 1

Choice a:

J,M, T/W/K

D,E,S

R, T/W/K, T/W/K

No violation. Hence this combination allowed

Choice b:

J,M, D/W/K/S

E,R,T

D/W/K/S, D/W/K/S, D/W/K/S

Violates because there would be no parent in group 3.

Hence the correct answer.

55. If each of the Henderson Children rides in a different canoe, which of the
following must be true?

I. The Penick children do not ride together

II. The Penick parents do not ride together,

III. The Henderson Parents do not ride together


a. Only I
b. Only II
c. I and II
d. I and III

Step 2: Use info in the question and in part(i).

T, K,S

Violation occurs in group 1.

Therefore Part (i) is true because penick children cannot ride together as the
group will be without a parent

Use info in the question and part (ii)

T,E,J

W,R,S

D,M,K

Need not be true because as we see the penick parents can ride together

Use info in question and in part (iii)

T,R,M

Violation occurs and therefore Part (iii) is true because if Henderson parents ride
together, the group will not have a member from Family 2.
Hence the correct answer is Choice d.

Questions 58-60

Each of 5 people –A,B,C,D,E owns a different car among Maruti, Mercedes, Sierra,
Fiat and Audi and the colors of these cars are Black, green, blue, white and red,
not necessarily in that order. No to cars are of the same color. It is also known
that

(i) A’s car is not black and it is not a Mercedes


(ii) B’s car is green and is not a Sierra
(iii) E’s car is not white and it is not an Audi.
(iv) C’s car is a Mercedes and it is not blue
(v) D’s car is not red and it is a Fiat.

Solution:

Step 1: Write down the facts and rules in the problem

A B C D E

~blk g `~blu ~r ~w

~mer ~s mer f ~a

58. If A owns a Blue Sierra, E’s car can be a ,

a. Red Maruti

b. White Maruti

c. Black Audi
d. Red Audi

Step 2: Use info in the question

Blu

Sie

Step 3: Use info in the problem

A B C D E

Blu g `~blu ~r ~w

Sie ~s mer f ~a

Step 4: Use info in the choices 1 by 1

Choice a:

A B C D E

Blu g `~blu ~r r

Sie ~s mer f ma

Does not violate any constraint. Hence is the correct answer

59. If a owns a white Audi, E’s car can be a

a. Red Maruti

b. Blue Maruti

c. Green Audi

d. Black Sierra

Step 2: Use info in the question

A
w

Step 3: Use info in the problem

A B C D E

w g `~blu ~r ~w

a ~s mer f ~a

Step 4: Use info in the choices 1 by 1

Choice a:

A B C D E

w g `~blu ~r r

a ~s mer f ma

Violates because B will own Maruti.

Choice b:

A B C D E

w g `~blu ~r blu

a ~s mer f ma

Same violation

Choice c:

A B C D E
w g `~blu ~r g

a ~s mer f a

Violates because B owns the green car

Choice d:

A B C D E

w g `~blu ~r blu

a ~s mer f s

No violation. So is the right answer.

60. If A’s car is a Red Maruti and D’s car is white, then E owns a

a. Black Audi

b. Blue Sierra

c.Black Sierra

d. Blue Audi

Step 2: Use info in the question

A B C D E

r w

mar

Step 3: Use info in the problem

A B C D E
r g `~blu w ~w

mar ~s mer f ~a

Step 4: Use info in the choices 1 by 1

Choice a:

A B C D E

r g `~blu w blk

mar ~s mer f a

Violates because e does not own an Audi

Choice b:

A B C D E

r g `~blu w blu

mar ~s mer f s

No violation and hence is the correct answer.

Вам также может понравиться